Prove that there exist $135$ consecutive positive integers so that the $n$th least is divisible by a perfect $n$th power greater than $1$product of six consecutive integers being a perfect squareCan both $x^2 + y+2$ and $y^2+4x$ be squares?$n mid k^2 land n+1 mid l^3 land n+2 mid m^4 to n=?$Set whose average of subsets is always square (cube, etc.)Prove that there exist two integers such that i - j is divisible by n.Number Theory Homework: Find 3 consecutive integers…Question from Mathcounts competitionProve that there are no positive integers $x$ and $y$ such that $x^3 + y^3 = 10^3$.odd integers that are divisible by a perfect cubeCan I complete the euclidean case without elliptic-curve-theory?

If the Dominion rule using their Jem'Hadar troops, why is their life expectancy so low?

Error in master's thesis, I do not know what to do

"Oh no!" in Latin

New Order #2: Turn My Way

Center page as a whole without centering each element individually

How to preserve electronics (computers, ipads, phones) for hundreds of years?

Why does a 97 / 92 key piano exist by Bosendorfer?

How would a solely written language work mechanically

Why didn't Voldemort know what Grindelwald looked like?

"Marked down as someone wanting to sell shares." What does that mean?

Started in 1987 vs. Starting in 1987

Offset in split text content

How to test the sharpness of a knife?

Sort with assumptions

What is the tangent at a sharp point on a curve?

Hashing password to increase entropy

Air travel with refrigerated insulin

When is the exact date for EOL of Ubuntu 14.04 LTS?

Reasons for having MCU pin-states default to pull-up/down out of reset

How do I lift the insulation blower into the attic?

How do you justify more code being written by following clean code practices?

C++ lambda syntax

Strange behavior in TikZ draw command

Checking @@ROWCOUNT failing



Prove that there exist $135$ consecutive positive integers so that the $n$th least is divisible by a perfect $n$th power greater than $1$


product of six consecutive integers being a perfect squareCan both $x^2 + y+2$ and $y^2+4x$ be squares?$n mid k^2 land n+1 mid l^3 land n+2 mid m^4 to n=?$Set whose average of subsets is always square (cube, etc.)Prove that there exist two integers such that i - j is divisible by n.Number Theory Homework: Find 3 consecutive integers…Question from Mathcounts competitionProve that there are no positive integers $x$ and $y$ such that $x^3 + y^3 = 10^3$.odd integers that are divisible by a perfect cubeCan I complete the euclidean case without elliptic-curve-theory?













1












$begingroup$


Prove that there exist 135 consecutive positive integers so that the second least is divisible by a perfect square $> 1$, the third least is divisible by a perfect cube $> 1$, the fourth least is divisible by a perfect fourth power $> 1$, and so on.



How should I go about doing this?



I thought perhaps I should use Fermat's little theorem, or its corollary?



Thanks!










share|cite|improve this question









New contributor




Sania is a new contributor to this site. Take care in asking for clarification, commenting, and answering.
Check out our Code of Conduct.







$endgroup$











  • $begingroup$
    Wow, this problem sounds painful.
    $endgroup$
    – Don Thousand
    21 hours ago















1












$begingroup$


Prove that there exist 135 consecutive positive integers so that the second least is divisible by a perfect square $> 1$, the third least is divisible by a perfect cube $> 1$, the fourth least is divisible by a perfect fourth power $> 1$, and so on.



How should I go about doing this?



I thought perhaps I should use Fermat's little theorem, or its corollary?



Thanks!










share|cite|improve this question









New contributor




Sania is a new contributor to this site. Take care in asking for clarification, commenting, and answering.
Check out our Code of Conduct.







$endgroup$











  • $begingroup$
    Wow, this problem sounds painful.
    $endgroup$
    – Don Thousand
    21 hours ago













1












1








1


2



$begingroup$


Prove that there exist 135 consecutive positive integers so that the second least is divisible by a perfect square $> 1$, the third least is divisible by a perfect cube $> 1$, the fourth least is divisible by a perfect fourth power $> 1$, and so on.



How should I go about doing this?



I thought perhaps I should use Fermat's little theorem, or its corollary?



Thanks!










share|cite|improve this question









New contributor




Sania is a new contributor to this site. Take care in asking for clarification, commenting, and answering.
Check out our Code of Conduct.







$endgroup$




Prove that there exist 135 consecutive positive integers so that the second least is divisible by a perfect square $> 1$, the third least is divisible by a perfect cube $> 1$, the fourth least is divisible by a perfect fourth power $> 1$, and so on.



How should I go about doing this?



I thought perhaps I should use Fermat's little theorem, or its corollary?



Thanks!







elementary-number-theory divisibility






share|cite|improve this question









New contributor




Sania is a new contributor to this site. Take care in asking for clarification, commenting, and answering.
Check out our Code of Conduct.











share|cite|improve this question









New contributor




Sania is a new contributor to this site. Take care in asking for clarification, commenting, and answering.
Check out our Code of Conduct.









share|cite|improve this question




share|cite|improve this question








edited 16 hours ago









user21820

39.6k543156




39.6k543156






New contributor




Sania is a new contributor to this site. Take care in asking for clarification, commenting, and answering.
Check out our Code of Conduct.









asked 21 hours ago









SaniaSania

314




314




New contributor




Sania is a new contributor to this site. Take care in asking for clarification, commenting, and answering.
Check out our Code of Conduct.





New contributor





Sania is a new contributor to this site. Take care in asking for clarification, commenting, and answering.
Check out our Code of Conduct.






Sania is a new contributor to this site. Take care in asking for clarification, commenting, and answering.
Check out our Code of Conduct.











  • $begingroup$
    Wow, this problem sounds painful.
    $endgroup$
    – Don Thousand
    21 hours ago
















  • $begingroup$
    Wow, this problem sounds painful.
    $endgroup$
    – Don Thousand
    21 hours ago















$begingroup$
Wow, this problem sounds painful.
$endgroup$
– Don Thousand
21 hours ago




$begingroup$
Wow, this problem sounds painful.
$endgroup$
– Don Thousand
21 hours ago










2 Answers
2






active

oldest

votes


















3












$begingroup$

Use the Chinese Remainder Theorem. Pick $134$ distinct primes. The perfect square is the square of the first, the cube is the cube of the second, and so on. All your moduli are distinct, so CRT guarantees a solution. If you use the smallest primes in order and $N$ is the least of your $135$ numbers, you have $N+1 equiv 0 pmod 2^2, N+2 equiv 0 pmod 3^3, N+3 equiv 0 pmod 5^4ldots$






share|cite|improve this answer









$endgroup$




















    3












    $begingroup$

    By the Chinese remainder theorem, there is an integer $n$ such that $nequiv -k (textmod p_k^k+1)$ for all $k=1,2,dots 134$, (where $p_k$ is the $k^textth$ smallest prime). Then $n, n+1, dots, n+134$ satisfy the required condition.






    share|cite|improve this answer









    $endgroup$












      Your Answer





      StackExchange.ifUsing("editor", function ()
      return StackExchange.using("mathjaxEditing", function ()
      StackExchange.MarkdownEditor.creationCallbacks.add(function (editor, postfix)
      StackExchange.mathjaxEditing.prepareWmdForMathJax(editor, postfix, [["$", "$"], ["\\(","\\)"]]);
      );
      );
      , "mathjax-editing");

      StackExchange.ready(function()
      var channelOptions =
      tags: "".split(" "),
      id: "69"
      ;
      initTagRenderer("".split(" "), "".split(" "), channelOptions);

      StackExchange.using("externalEditor", function()
      // Have to fire editor after snippets, if snippets enabled
      if (StackExchange.settings.snippets.snippetsEnabled)
      StackExchange.using("snippets", function()
      createEditor();
      );

      else
      createEditor();

      );

      function createEditor()
      StackExchange.prepareEditor(
      heartbeatType: 'answer',
      autoActivateHeartbeat: false,
      convertImagesToLinks: true,
      noModals: true,
      showLowRepImageUploadWarning: true,
      reputationToPostImages: 10,
      bindNavPrevention: true,
      postfix: "",
      imageUploader:
      brandingHtml: "Powered by u003ca class="icon-imgur-white" href="https://imgur.com/"u003eu003c/au003e",
      contentPolicyHtml: "User contributions licensed under u003ca href="https://creativecommons.org/licenses/by-sa/3.0/"u003ecc by-sa 3.0 with attribution requiredu003c/au003e u003ca href="https://stackoverflow.com/legal/content-policy"u003e(content policy)u003c/au003e",
      allowUrls: true
      ,
      noCode: true, onDemand: true,
      discardSelector: ".discard-answer"
      ,immediatelyShowMarkdownHelp:true
      );



      );






      Sania is a new contributor. Be nice, and check out our Code of Conduct.









      draft saved

      draft discarded


















      StackExchange.ready(
      function ()
      StackExchange.openid.initPostLogin('.new-post-login', 'https%3a%2f%2fmath.stackexchange.com%2fquestions%2f3153661%2fprove-that-there-exist-135-consecutive-positive-integers-so-that-the-nth-lea%23new-answer', 'question_page');

      );

      Post as a guest















      Required, but never shown

























      2 Answers
      2






      active

      oldest

      votes








      2 Answers
      2






      active

      oldest

      votes









      active

      oldest

      votes






      active

      oldest

      votes









      3












      $begingroup$

      Use the Chinese Remainder Theorem. Pick $134$ distinct primes. The perfect square is the square of the first, the cube is the cube of the second, and so on. All your moduli are distinct, so CRT guarantees a solution. If you use the smallest primes in order and $N$ is the least of your $135$ numbers, you have $N+1 equiv 0 pmod 2^2, N+2 equiv 0 pmod 3^3, N+3 equiv 0 pmod 5^4ldots$






      share|cite|improve this answer









      $endgroup$

















        3












        $begingroup$

        Use the Chinese Remainder Theorem. Pick $134$ distinct primes. The perfect square is the square of the first, the cube is the cube of the second, and so on. All your moduli are distinct, so CRT guarantees a solution. If you use the smallest primes in order and $N$ is the least of your $135$ numbers, you have $N+1 equiv 0 pmod 2^2, N+2 equiv 0 pmod 3^3, N+3 equiv 0 pmod 5^4ldots$






        share|cite|improve this answer









        $endgroup$















          3












          3








          3





          $begingroup$

          Use the Chinese Remainder Theorem. Pick $134$ distinct primes. The perfect square is the square of the first, the cube is the cube of the second, and so on. All your moduli are distinct, so CRT guarantees a solution. If you use the smallest primes in order and $N$ is the least of your $135$ numbers, you have $N+1 equiv 0 pmod 2^2, N+2 equiv 0 pmod 3^3, N+3 equiv 0 pmod 5^4ldots$






          share|cite|improve this answer









          $endgroup$



          Use the Chinese Remainder Theorem. Pick $134$ distinct primes. The perfect square is the square of the first, the cube is the cube of the second, and so on. All your moduli are distinct, so CRT guarantees a solution. If you use the smallest primes in order and $N$ is the least of your $135$ numbers, you have $N+1 equiv 0 pmod 2^2, N+2 equiv 0 pmod 3^3, N+3 equiv 0 pmod 5^4ldots$







          share|cite|improve this answer












          share|cite|improve this answer



          share|cite|improve this answer










          answered 21 hours ago









          Ross MillikanRoss Millikan

          299k24200374




          299k24200374





















              3












              $begingroup$

              By the Chinese remainder theorem, there is an integer $n$ such that $nequiv -k (textmod p_k^k+1)$ for all $k=1,2,dots 134$, (where $p_k$ is the $k^textth$ smallest prime). Then $n, n+1, dots, n+134$ satisfy the required condition.






              share|cite|improve this answer









              $endgroup$

















                3












                $begingroup$

                By the Chinese remainder theorem, there is an integer $n$ such that $nequiv -k (textmod p_k^k+1)$ for all $k=1,2,dots 134$, (where $p_k$ is the $k^textth$ smallest prime). Then $n, n+1, dots, n+134$ satisfy the required condition.






                share|cite|improve this answer









                $endgroup$















                  3












                  3








                  3





                  $begingroup$

                  By the Chinese remainder theorem, there is an integer $n$ such that $nequiv -k (textmod p_k^k+1)$ for all $k=1,2,dots 134$, (where $p_k$ is the $k^textth$ smallest prime). Then $n, n+1, dots, n+134$ satisfy the required condition.






                  share|cite|improve this answer









                  $endgroup$



                  By the Chinese remainder theorem, there is an integer $n$ such that $nequiv -k (textmod p_k^k+1)$ for all $k=1,2,dots 134$, (where $p_k$ is the $k^textth$ smallest prime). Then $n, n+1, dots, n+134$ satisfy the required condition.







                  share|cite|improve this answer












                  share|cite|improve this answer



                  share|cite|improve this answer










                  answered 21 hours ago









                  Poon LeviPoon Levi

                  48638




                  48638




















                      Sania is a new contributor. Be nice, and check out our Code of Conduct.









                      draft saved

                      draft discarded


















                      Sania is a new contributor. Be nice, and check out our Code of Conduct.












                      Sania is a new contributor. Be nice, and check out our Code of Conduct.











                      Sania is a new contributor. Be nice, and check out our Code of Conduct.














                      Thanks for contributing an answer to Mathematics Stack Exchange!


                      • Please be sure to answer the question. Provide details and share your research!

                      But avoid


                      • Asking for help, clarification, or responding to other answers.

                      • Making statements based on opinion; back them up with references or personal experience.

                      Use MathJax to format equations. MathJax reference.


                      To learn more, see our tips on writing great answers.




                      draft saved


                      draft discarded














                      StackExchange.ready(
                      function ()
                      StackExchange.openid.initPostLogin('.new-post-login', 'https%3a%2f%2fmath.stackexchange.com%2fquestions%2f3153661%2fprove-that-there-exist-135-consecutive-positive-integers-so-that-the-nth-lea%23new-answer', 'question_page');

                      );

                      Post as a guest















                      Required, but never shown





















































                      Required, but never shown














                      Required, but never shown












                      Required, but never shown







                      Required, but never shown

































                      Required, but never shown














                      Required, but never shown












                      Required, but never shown







                      Required, but never shown







                      Popular posts from this blog

                      Sum ergo cogito? 1 nng

                      419 nièngy_Soadمي 19bal1.5o_g

                      Queiggey Chernihivv 9NnOo i Zw X QqKk LpB